0
$\begingroup$

Let $A$ be an $n\times n$ symmetric matrix and $S$ be an $n\times n$ positive definite matrix. Suppose the eigenvalues of $A$ and $SAS$ are arranged in non decreasing order; that is $$ \lambda_1 (A) \leq \cdots \leq \lambda_n (A)\; \mbox{and}\; \lambda_1 (SAS) \leq \cdots \leq \lambda_n (SAS). $$ Assume that $\lambda_1 (A) + \cdots + \lambda_k (A) > 0$. Does it hold or not that $$ \lambda_{i_1} (SAS) + \cdots + \lambda_{i_k} (SAS) = \theta_{i_1\cdots i_k} (\lambda_{i_1} (A) + \cdots + \lambda_{i_k} (A)) $$ for some positive $\theta_{i_1\cdots i_k} $?

$\endgroup$
1
  • 3
    $\begingroup$ Why would you suspect this to be true? The $n$-sum would be a trace, and that of $SAS$ is basically a weighted sum. So take $A = \mathrm{diag}(-1,2)$ and $S = \mathrm{diag}(10,1)$ then $SAS = \mathrm{diag}(-100,2)$. You have that $A$ has positive trace but $SAS$ has negative trace. $\endgroup$ Commented Oct 20 at 18:25

1 Answer 1

2
$\begingroup$

$\newcommand\la\lambda$The answer is no. E.g., take $n=3$, $$A=\left( \begin{array}{ccc} -1 & 0 & 0 \\ 0 & 2 & 0 \\ 0 & 0 & 2 \\ \end{array} \right),\quad S=XX^\top,\quad X=\left( \begin{array}{ccc} -10 & 10 & -10 \\ 7 & 3 & -10 \\ -3 & -10 & 10 \\ \end{array} \right),$$ $k=2$, $i_1=1$, and $i_2=2$, so that $$\la_{i_1}(A)+\dots+\la_{i_k}(A) =\la_1(A)+\dots+\la_k(A)=-1+2=1>0,$$ whereas $$\la_{i_1}(SAS)+\dots+\la_{i_k}(SAS)\approx -44980+283<0.$$ So, there is no positive $\theta_{i_1,\dots,i_k}$ such that $$\la_{i_1}(SAS)+\dots+\la_{i_k}(SAS)=\theta_{i_1,\dots,i_k} (\la_{i_1}(A)+\dots+\la_{i_k}(A)).$$

$\endgroup$

You must log in to answer this question.

Start asking to get answers

Find the answer to your question by asking.

Ask question

Explore related questions

See similar questions with these tags.